• Shuffle
    Toggle On
    Toggle Off
  • Alphabetize
    Toggle On
    Toggle Off
  • Front First
    Toggle On
    Toggle Off
  • Both Sides
    Toggle On
    Toggle Off
  • Read
    Toggle On
    Toggle Off
Reading...
Front

Card Range To Study

through

image

Play button

image

Play button

image

Progress

1/101

Click to flip

Use LEFT and RIGHT arrow keys to navigate between flashcards;

Use UP and DOWN arrow keys to flip the card;

H to show hint;

A reads text to speech;

101 Cards in this Set

  • Front
  • Back
RH is a 50 year old Caucasian man with a history of hypertension and Type 2 diabetes mellitus. He comes to the clinic today with complaints of pain in the chest area that is “squeezing” in character when he does daily chores such as mowing the lawn. The pain is 7 on a scale of 1 to 10, starts with exertion and is relieved by rest. His father died of a heart attack when he was 70 and his mother has hypertension. He has had both hypertension and diabetes for over 3 years, and his diabetes is well-controlled by diet and medication. His cardiologist wants to treat his ischemic heart disease medically.
Meds:
metformin XR 1000 mg daily
ramipril 10 mg daily
VS: BP 135/68 P75 RR22 T36.4
Labs: HbA1c: 6.8, other labs within normal limits
Diagnosis: Chronic stable angina

Which of the following changes to RH’s drug regimen would be BEST for initial angina symptom control in this patient?
a. Add atenolol, sublingual nitroglycerin, and verapamil
b. Add sublingual nitroglycerin and increase dose of ramipril
c. Add atenolol and amlodipine
d. Add atenolol and sublingual nitroglycerin
e. Add verapamil and sublingual nitroglycerin
d. Add atenolol and sublingual nitroglycerin

A is incorrect because we would rarely use atenolol and verapamil together (potential AV block, etc) and because one would not add 3 drugs at once. B does not offer sufficient prophylaxis of angina symptoms in this patient. C is incorrect because we would not initiate both of these drugs (atenolol and amlodipine) at the same time unless we found that atenolol alone was not sufficient to control his symptoms. E is incorrect since he does not have contraindications to a beta blocker (glycemia is under control), first line therapy should be a beta blocker, so this is not the best answer. Patients should also have SL NTG available to them for relief of acute attacks.
Refer to Case A
This patient is considered by the medical team to have “high-risk” features for subsequent adverse outcomes due to ACS, and have decided to treat him with aggressive medical management right away. Which of the following is the most significant high-risk feature that this patient exhibits?
a. Tachycardia in the setting of advanced age
b. Positive troponin
c. Accelerating tempo of ischemic symptoms in the preceding 48 hours
d. Signs of pulmonary edema
e. Two of the above
b. Positive troponin

A-Incorrect. Both tachycardia and Age>75 years are considered high risk features; however, this patient has neither.
B- Correct. Positive troponin is considered a high risk feature, and this patient’s cardiac enzymes reveal positive troponin.
C- Incorrect. While this is a high risk feature, the patient has not had this feature based on presentation.
D- Incorrect. While signs of heart failure are considered high risk features, this patient’s lungs are clear on physical exam.
E-Incorrect. Only B is the correct answer.
Refer to Case A
The medical team initiates aspirin and anti-thrombin therapy. Which of the following is the most appropriate intervention in this patient and why?
a. Enoxaparin because it has been shown to be superior to heparin in this type of ACS
b. Heparin because this patient has renal dysfunction that precludes him from
receiving enoxaparin
c. Fondaparinux because this patient is at sufficient bleeding risk to justify
this agent
d. Bivalirudin because it has been shown to have superior net clinical benefit
in ACS
e. None of the rationales are correct
a. Enoxaparin because it has been shown to be superior to heparin in this type of ACS

A- Correct. Data from studies such as ESSENCE, TIMI 11B, and INTERACT demonstrated superiority of enoxaparin over heparin on endpoints including the composite for death/MI/need for revascularization in NSTE ACS.
B-Incorrect. This patient has impaired renal function. However, this doesn’t preclude therapy with enoxaparin. In addition, reduced doses of enoxaparin can be used for treatment of NSTE and STE ACS according to the product labeling.
C- Incorrect. Fondaparinux is not superior to enoxaparin for clinical endpoint reduction. While fondaparinux has a better bleeding risk profile, this patient is not at a high risk of bleeding.
D-Incorrect. Bivalirudin is reserved as an alternative to heparin or enoxaparin in the cath lab/PCI, not for medical management of ACS.
E-Incorrect. A is correct.
Refer to Case A
Which of the following medications would be inappropriate in this patient?
a. Tenecteplase
b. Abciximab
c. Clopidogrel
d. A and B
e. A and C
d. A and B

Clopidogrel is clearly warranted. It is indicated in conjunction with ASA for the medical management of people with NSTE ACS, unless they have specific contraindications. Lytics such as tenecteplase are contraindicated in people with NSTE ACS. Also, abciximab is not indicated for medical management of NSTE ACS based on the neutral effects (and perhaps trends toward harm) seen in the GUSTO-V study. Only eptifibatide and tirofiban are indicated for medical management of NSTE ACS.
Refer to Case A
The patient is admitted and will be transferred to a nearby hospital for Percutaneous Coronary Intervention (PCI)/stent placement tomorrow. He was given a 300 mg oral dose of clopidogrel on admission. Which of the following is most appropriate regarding this person’s clopidogrel?
a. Clopidogrel is not appropriate in this patient and should not be given again
b. The patient should be given an additional 300 mg dose before being
transferred on the day of PCI
c. The patient should be given 75 mg before being transferred on the day of PCI
d. None of the above are correct
A- Incorrect. Clopidogrel is indicated in conjunction with ASA for the medical management of people with NSTE ACS as well as post stent placement, unless they have specific contraindications which this patient does not have.
B- Incorrect. The 300 mg he received on admission is sufficient since he is going to the cath lab >6hrs after the dose.
C is correct. The patient should be given 75 mg daily and will need a dose in the am. Correct. Clopidogrel 75 mg daily is indicated in patients post-PCI with stent. The type of stent will dictate the duration of treatment.
Refer to Case A
The medical team has decided to initiate eptifibatide. Which of the following is the appropriate regimen in this patient based on the prescribing information provided at the end of this exam?
a. Bolus of 8.5 ml; infusion of 15 ml/hr
b. Bolus of 8.5 ml; infusion of 15 ml/kg/hr
c. Bolus of 4.25 ml; infusion of 7.5 ml/hr
d. Eptifibatide is not appropriate in this patient and should not be given
e. None of the above is correct
e. None of the above is correct

A, B, and C are incorrect. Incorrect. The dose should be 8.5 ml bolus and renally adjusted infusion is 7.5 ml/hr. D is incorrect because this person has NSTEMI with high risk features, making medical management with a GPIIBIIIA inhibitor like tirofiban or eptifibatide acceptable. Thus, E is the correct answer.
Refer to Case A
The physician is impressed by your ability to take care of this patient with ACS. He wants to ask you a side question on one of his other patients with ACS who is going to the cath lab. The physician tells you that the patient is on enoxaparin and received their last dose 4 hours ago. He wants to know if you think the patient should get another dose of enoxaparin now on his way to the cath lab. You answer with which of the following?
a. No; the patient doesn’t need another dose unless he received his last enoxaparin dose more than 8 hours ago
b. Yes; the patient needs another dose since he received his last enoxaparin dose more than 2 hours ago
c. It’s up to the physician; there is no good evidence either way
d. No; the patient should not get another dose, but should get bivalirudin in the cath lab instead
e. None of the above is correct
a. No; the patient doesn’t need another dose unless he received his last enoxaparin dose more than 8 hours ago

A- Correct. If a patient on enoxaparin has received their dose within 8 hours of going to the cath lab, they don’t need a supplemental dose.
B- Incorrect; patients only need supplemental doses if they received their last dose at least 8 hours ago.
C- Incorrect. There is evidence that a supplemental dose for patients who have received their last dose at least 8 hours ago improves anti-coagulation
D- Incorrect. Bivalirudin is generally reserved as alternative to heparin/enoxaparin in patients at high bleeding risk or allergies.
E- Incorrect. A is correct.
Refer to Case B
Which of the following physical findings in this patient are most consistent with the diagnosis of heart failure?
a. Elevated jugular venous pressure, third heart sound, and displaced apical pulse
b. Edema, fever, hepatomegaly, and fatigue
c. Tachycardia, rales, elevated white blood cell count
d. Cardiomegaly, constipation, myopathy
a. Elevated jugular venous pressure, third heart sound, and displaced apical pulse

Elevated jugular venous pressure, third heart sound, and displaced apical pulse are all cardinal signs of heart failure found on physical exam. Fever, elevated white blood cell count, and myopathy are not signs of heart failure.
Refer to Case B
Which of the following most accurately describes the role of diuretics in heart
failure?
a. Diuretics are a mainstay of heart failure treatment and should be used as
monotherapy in asymptomatic patients
9
b. Diuretics decrease sympathetic outflow and reduce activation of the renin-
angiotenin-aldosterone system
c. Diuretics are a mainstay of heart failure treatment because of their ability to reduce volume overload and provide symptomatic relief
d. Diuretics should be avoided in heart failure because of their ability to cause volume depletion and worsen renal function
c. Diuretics are a mainstay of heart failure treatment because of their ability to reduce volume overload and provide symptomatic relief

Answer A is not correct because even asymptomatic patients should be treated with ACE inhibitors (stage A for treating risk factors, and all stage B, C, and D HF patients). Answer B is not correct because diuretics INCREASE sympathetic outflow and can activate the RAAS. D is incorrect because diuretics are used to improve symptoms of fluid overload. It is true however that over-diuresis should be avoided so that volume depletion and worsened renal function can be avoided. Answer C is correct because diuretics DO provide symptomatic relief by reducing volume overload.
Refer to Case B
Which of the following would be the most appropriate short-term change to CB’s
heart failure medication regimen?
a. Discontinue propranolol and start carvedilol 3.125 mg bid
b. Increase HCTZ dose to 50 mg QD
c. Change HCTZ to furosemide 40 mg BID
d. Start spironolactone 25 mg QD
c. Change HCTZ to furosemide 40 mg BID

This patient is clearly fluid overloaded and experiencing an exacerbation. This is the first priority in this patient. Thiazide diuretics are not potent enough diuretics to diuresis a patient in this situation and increasing HCTZ dose will not be adequate. Potassium-sparing diuretics are also not potent enough diuretics for an acute exacerbation. Answer A should be done, but at a later date when the patient is euvolemic. Beta-blockers should not be initiated or titrated while patients are unstable or fluid overloaded.
Refer to Case B
Which of the following additions to CB’s heart failure regimen should be made
first?
a. candesartan 4 mg QD
b. metolazone 5mg QD
c. digoxin 0.125 mg QD
d. lisinopril 10 mg QD
d. lisinopril 10 mg QD

This patient does not have any apparent contraindications to ACE inhibitor therapy and this is the top priority. ACE inhibitors are preferred over ARBs because of they have more data behind their use. Metalozone’s primary role is in combination with loop diuretics in resistant patients and this patient has not even been tried on furosemide yet. Digoxin does reduce hospitalizations and improve symptoms, but it does not improve mortality. Digoxin’s role in HF is for patient’s who remain symptomatic on optimum therapy. This patient is not on ACE inhibitor or appropriate beta-blocker therapy yet.
Refer to Case B
Which of the following changes to CB’s non-heart failure regimen would be most beneficial to this patient?
a. Increase ibuprofen to 800 mg TID
b. Start Vitamin E 1000 IU QD
c. Discontinue ranitidine 150 mg QHS
d. Change rosiglitazone 8mg QD to glipizide 10mg QD
d. Change rosiglitazone 8mg QD to glipizide 10mg QD
Which of the following represents a correct statement regarding the treatment of
heart failure?
a. Hydralazine/isosorbide dinitrate should be substituted for ACEIs in
African Americans
b. Beta-blockers should be initiated when patients are euvolemic and on
stable diuretic doses
c. Angiotensin receptor blockers should be added to regimens when patients are on stable dose ACE inhibitor, beta-blocker, and spironolactone
d. Digoxin should be withdrawn prior to the initiation of
hydralazine/isosorbide dinitrate
b. Beta-blockers should be initiated when patients are euvolemic and on
stable diuretic doses
Which of the following scenarios of ACE inhibitor intolerance in a heart failure
patient would it be most appropriate to switch to an angiotensin receptor blocker?
a. Patient who develops renal dysfunction with ACE inhibitor treatment
b. Patient who develops hyperkalemia with ACE inhibitor treatment
c. Patient who develops hypotension with ACE inhibitor treatment
d. Patient who develops intolerable cough with ACE inhibitor treatment
d. Patient who develops intolerable cough with ACE inhibitor treatment

Renal dysfunction, hyperkalemia, and hypotension are all side effects of ACE inhibitors and ARBs. Cough is only a problem with ACE inhibitors, not ARBs because ACE inhibitors increase bradykinin and ARBs don’t.
Which of the following is NOT a contraindication to spironolactone therapy?
a. Potassium < 3.5 mmol/L
b. Serum creatinine > 2.5 mg/dL
c. Potassium > 5.0 mmol/L
d. Recent (within 6 months) HF exacerbation
e. A and D
e. A and D

The contraindications to spironolactone based on the RALES study are serum creatinine > 2.5 mg/dL and potassium > 5.0 mmol/L. Class IV HF or recent HF exacerbation were inclusion criteria in the RALES trial. Spironolactone can increase potassium concentrations so a patient with low potassium could benefit from treatment with it.
MY is a 34 year-old black woman with well-controlled hypertension. She comes
to the pharmacy requesting a refill of labetalol and a new prescription from her
obstetrician for prenatal vitamins. Which of the following is most correct?
a. labetalol is acceptable during pregnancy, refill the prescription and
counsel her on the potential risks
b. valsartan is more effective in pregnancy, refill the prescription but
recommend that she seek a new prescription for valsartan
c. only methyldopa is safe during pregnancy, do not refill the prescription
and call the doctor to change the medication
d. no drugs are safe during pregnancy, do not refill the prescription and
recommend sodium restriction
a. labetalol is acceptable during pregnancy, refill the prescription and
counsel her on the potential risks

A- Correct because labetalol is safer option for managing chronic hypertension in pregnancy.
B- Incorrect because ACEI and ARBs are absolutely contraindicated in pregnant women.
C- Incorrect because methyldopa, while commonly used by OB/GYNs and safe during pregnancy, other medications including beta-blockers.
D- Incorrect because hypertension can be treated pharmacologically in pregnancy as above.
LM is a 34 year-old obese white woman with diabetes and newly-diagnosed hypertension (stage 2). Which of the following most appropriately applies to the clinical management of this patient?
a. 6-12 weeks of lifestyle modification should be implemented prior to
starting the DASH diet
b. The DASH diet should be implemented immediately as monotherapy
because it will decrease her blood pressure by 8-14 mmHg
c. Start monotherapy antihypertensive therapy
d. Start combination antihypertensive therapy with two drugs from
different classes
d. Start combination antihypertensive therapy with two drugs from
different classes

A-Incorrect. Drug therapy is warranted immediately and should be given concomitant with lifestyle modification, including the DASH diet
12
B-Incorrect because DASH is a dietary program that encourages nutrient-rich foods, less fats and cholesterol, higher fiber, lower sodium. Drug therapy is warranted immediately and should be given concomitant with lifestyle modification, including the DASH diet
C- Incorrect because LM has stage 2 hypertension and monotherapy is unlikely to lower BP to her goal of <130/80
D- Correct because stage 2 hypertension would place LM in a category where her BP is >160/>100. If her goal as a diabetic is <130/<80, she is >20/>10 from her goal by definition and would benefit from starting on 2 drugs.
JT is 45 year-old man that returns for follow-up 3 months later after starting Drug
A for newly diagnosed hypertension. After home blood pressure monitoring
during the first month of treatment, the physician maximized the dose of Drug A.
His BP is still elevated at 152/98 mmHg although he had demonstrated a response
(systolic is 5-10 mmHg lower) to Drug A. Which of the following is the most correct next step in this patient’s therapy?
a. discontinue Drug A and start a different drug from the same class
b. discontinue Drug A and start a different drug from a different class
c. continue Drug A and add a drug from a different class
d. continue drug Drug A and add two drugs from different classes
c. continue Drug A and add a drug from a different class

A- Incorrect because the likelihood of deriving greater benefit from a drug within the same class is unlikely.
B- Incorrect because the patient demonstrated some response to the initial drug choice. A second drug should be added to achieve the goal.
C-Correct because adding a drug that targets a different pathophysiological component is the most logical approach to reducing the patient’s blood pressure.
D-Incorrect because adding a single drug may be enough to achieve his goal of <140/<90. A third drug may be considered if he does not demonstrate a response to the 2 drug regimen at maximal doses.
MS, a 50 year-old woman with essential hypertension who is otherwise in good
health, comes to the pharmacy to refill her HCTZ, amlodipine, and ramipril. She
checks her blood pressure at the machine near the pharmacy (which was recently
calibrated and is accurate). She tells you her blood pressure was 185/112. She
feels well and there is no evidence of target organ damage. Which of the fllowing is most appropriate at this time?
a. call 911
b. advise her to see her physician within 1 week or go to the emergency
room
c. advise her to see her physician within 1 month
d. advise her to double the dose of amlodipine until her next appointment
b. advise her to see her physician within 1 week or go to the emergency
room

A- Incorrect because she is not symptomatic (e.g. having a myocardial infarction, etc.)
B- Correct because she is in need of urgent care, but not necessarily emergent.
C- Incorrect because her level of blood pressure is high enough to raise concerns about complications.
D-Incorrect because, beyond being a liability issue in recommending a dose change, she should be seen within the week.
KN is a 67 year-old man with longstanding uncomplicated essential
hypertension. He takes hydrochlorothiazide 25 mg daily. He has had a
reasonably good response to hydrochlorothiazide, but his home blood pressures
have been above goal, ranging from145-155/95-105 mmHg during the past several weeks. Which of the following would be the most logical modification to KN’s existing antihypertensive therapy?
a. increase hydrochlorothiazide to 50 mg daily
b. changing hydrochlorothiazide to atenolol 25 mg daily
c. adding amlodipine 5 mg daily to hydrochlorothiazide
d. adding lisinopril 10 mg daily to hydrochlorothiazide
d. adding lisinopril 10 mg daily to hydrochlorothiazide

A-Incorrect because increasing the dose of HCTZ from 25 mg to 50 mg seldom produces substantial decreases in blood pressure.
B-Incorrect because he has had a good response to hydrochlorothiazide so it would be better to add a drug to achieve additional decreases in blood pressure
C-Incorrect because the logical addition to hydrochlorothiazide would be an ACEI or ARB (slide 26 from lecture)).
D-Correct because adding a drug from a distinct class that targets renin activity is the most logical (slide 26 from lecture), and the combination of and ACEI or ARB with a diuretic tends to be very effective.
You are rounding with the internal medicine team, who is preparing to discharge
JS, an 84 year-old woman admitted with hip fracture secondary to fall. She also
has isolated systolic hypertension (average blood pressures 135-145/75-85 mmHg) and early Alzheimer’s. Her discharge medication list is as follows:
acetaminophen 300 mg/ codeine 30 mg one tablet as needed
alendronate 70 mg q Sunday
lansoprazole 30 mg daily
transdermal clonidine 0.1 mg/day apply weekly
hydrochlorothiazide 25 mg daily

Which of the following is the best recommendation regarding the management of her hypertension:
a. Eliminate codeine from the analgesic regimen because it increases blood
pressure
b. change clonidine to ramipril because clonidine causes orthostasis
c. decrease hydrochlorothiazide to 12.5 mg to reduce risk for further
cognitive decline
d. discontinue all antihypertensive drugs because treating hypertension in the
very elderly is not necessary
b. change clonidine to ramipril because clonidine causes orthostasis

A-Incorrect because codeine does not increase blood pressure
B-Correct because clonidine can cause orthostasis, and she will likely need to continue on a 2 drug regimen. Ramipril is synergistic with hydrochlorothiazide.
C-Incorrect because hydrochlorothiazide is not associated worsening of Alzheimer’s disease
D-Incorrect because treating hypertension, even in the very elderly, is beneficial
PT is a 42 year-old white man with hypertension and dyslipidemia presenting for
follow-up with his primary care physician 1 week after being discharged for
myocardial infarction. He is currently receiving ramipril 10 mg daily, aspirin 81
mg daily, and atorvastatin 40 mg daily. His blood pressure is 149/80 today. Which of the following is the most appropriate recommendation for this patient at this time?
a. Starting carvedilol
b. Increasing ramipril dose
c. Changing ramipril to hydrochlorothiazide
d. Starting verapamil
a. Starting carvedilol

A-Correct because myocardial infarction is a compelling indication for beta-blocker therapy
B-Incorrect because a beta-blocker should be added. Increasing the ramipril dose may be considered
C-Incorrect because the ACE inhibitor is indicated in post-MI patients
D-Incorrect because the evidence supports adding a beta-blocker rather than a nondihydropyridine CCB in post-MI patients in the absence of contraindications to beta-blockade
JT is a 48 year-old white woman with essential hypertension. Her blood pressure
has been controlled for the past 2 years on hydrochlorothiazide 25 mg daily. She
complains to you about the fact that she always has to leave her desk to urinate.
She wants to switch medications. Which of the following would be the best
alternative?
a. amlodipine
b. atenolol
c. irbesartan
d. lisinopril
a. amlodipine

A is correct because switching to a drug that targets a similar pathophysiology would be the most logical. Angiotensin receptor blockers, ACE inhibitors and beta-blockers address renin/ sympathetic related pathology, whereas diuretics and CCBs target peripheral resistance.
Refer to Case C
Which of the following are signs and symptoms of pulmonary congestion?
a. +jugular venous distension, orthopnea, rales
b. Weight gain, 2+ pedal edema, shortness of breath
c. Shortness of breath, fatigue, orthopnea, rales
d. Shortness of breath, orthopnea, rales
d. Shortness of breath, orthopnea, rales

JVD and pedal edema are signs of systemic congestion and fatigue is a general symptom, likely due to a combination of low cardiac output, fluid congestion, inadequate oxygen to skeletal muscles due to peripheral vasocontriction, among others.
Refer to Case C
Regarding therapies with the potential to slow TK’s disease process and prolong her survival, which of the following is the most appropriate recommendation?
a. add carvedilol once acute fluid overload corrected
b. increase lisinopril to 10 mg daily now, and add carvedilol once acute fluid overload corrected
c. increase lisinopril to 10 mg daily now, add digoxin now, and add carvedilol once acute fluid overload is corrected
d. increase lisinopril to 10 mg daily now and furosemide to 40 mg daily now, and add carvedilol once acute fluid overload is corrected
b. increase lisinopril to 10 mg daily now, and add carvedilol once acute fluid overload corrected

b is more correct than a since the lisinopril dose is not at the target survival dose and β-blocker needs added. c and d are incorrect because digoxin and furosemide do not have survival benefits.
Refer to Case C
Regarding approaches to improve her heart failure symptoms, which of the following would be the most appropriate recommendation now?
a. increase furosemide to 40 mg daily and add digoxin 0.125 mg daily
b. increase furosemide to 40 mg daily, increase lisinpril to 10 mg daily, add digoxin 0.125 mg daily
c. increase furosemide to 40 mg daily, increase lisinopril to 10 mg daily, discontinue celecoxib and replace with acetaminophen
d. increase furosemide to 40 mg daily, discontinue celecoxib and replace with acetaminophen.
c. increase furosemide to 40 mg daily, increase lisinopril to 10 mg daily, discontinue celecoxib and replace with acetaminophen
a and b are incorrect because digoxin should not be added until the other therapies are optimized. c is correct because furosemide dose needs acutely increased to get off the excess fluid, lisinopril needs increased for both symptomatic benefits and to get to target survival dose. Additionally the therapy that likely caused the current exacerbation should be stopped and therapy attempted with APAP. d is incorrect because it does not include a dose increase of lisinopril, which will improve symptoms.
LK is a 52 year old Hispanic male with a history of hyperlipidemia, type 2 diabetes mellitus, hypertension, and heart failure. His heart failure symptoms are currently classified as Class III. His current medications include atorvastatin 80 mg po qd, nateglinide 120 mg tid, acarbose 50 mg tid, lisinopril 10 mg qd, furosemide 60 mg qd, digoxin 0.25 mg qd. Current labs are as follows: fasting blood glucose 130 mg/dl, HbA1c 7.6%, Na – 135 mEq/L, K – 4.3 mEq/L, CL – 99 mEq/L, BUN 16 mg/dl, SCr 1.0 mg/dl, digoxin 1.5 ng/ml. His BP is 140/88 mmHg and pulse is 90/min.

The physician wishes to start rosiglitazone in this patient. Which of the following is the best assessment of this plan?
a. Additional diabetes therapy is not warranted in this patient at this time.
b. Additional therapy is warranted, but pioglitazone would be preferred in this patient due to his heart failure.
c. Rosiglitazone is considered contraindicated in this patient due to its propensity to cause edema and worsening heart failure.
d. Rosiglitazone is considered contraindicated in this patient due to its negative inotropic effects.
c. Rosiglitazone is considered contraindicated in this patient due to its propensity to cause edema and worsening heart failure.

a is incorrect as both the FBG and HgA1C indicate poor glucose control. b is incorrect because both rosiglitazone and pioglitazone are contraindicated. d is incorrect because this does not properly state the mechanism of detriment of these drugs in HF.
LK is a 52 year old Hispanic male with a history of hyperlipidemia, type 2 diabetes mellitus, hypertension, and heart failure. His heart failure symptoms are currently classified as Class III. His current medications include atorvastatin 80 mg po qd, nateglinide 120 mg tid, acarbose 50 mg tid, lisinopril 10 mg qd, furosemide 60 mg qd, digoxin 0.25 mg qd. Current labs are as follows: fasting blood glucose 130 mg/dl, HbA1c 7.6%, Na – 135 mEq/L, K – 4.3 mEq/L, CL – 99 mEq/L, BUN 16 mg/dl, SCr 1.0 mg/dl, digoxin 1.5 ng/ml. His BP is 140/88 mmHg and pulse is 90/min.

The physician also raises the issue of starting a B-blocker in this patient. Which of the following is the most appropriate response?
a. While B-blockers are beneficial in heart failure, it would be contraindicated in this patient because of his diabetes.
b. B-blocker is strongly indicated, and therapy should be initiated with carvedilol 6.25 mg bid.
c. B-blocker is strongly indicated, and therapy should be initiated with Toprol XL 12.5 mg qd
d. B-blocker should be started, but discontinued if the patient has any adverse changes in their lipid or glucose parameters.
c. B-blocker is strongly indicated, and therapy should be initiated with Toprol XL 12.5 mg qd

a is incorrect because β-blockers should not be withheld in diabetic HF patients. b is a suboptimal choice because the starting dose is not right. Additionally one might argue that in a patient with poor glucose control initiation of a β1-selective agent would be preferred over a nonselective agent. d is incorrect because the survival benefits are so great as to outweigh any changes the drug may cause in the lipid or glucose control.
LK is a 52 year old Hispanic male with a history of hyperlipidemia, type 2 diabetes mellitus, hypertension, and heart failure. His heart failure symptoms are currently classified as Class III. His current medications include atorvastatin 80 mg po qd, nateglinide 120 mg tid, acarbose 50 mg tid, lisinopril 10 mg qd, furosemide 60 mg qd, digoxin 0.25 mg qd. Current labs are as follows: fasting blood glucose 130 mg/dl, HbA1c 7.6%, Na – 135 mEq/L, K – 4.3 mEq/L, CL – 99 mEq/L, BUN 16 mg/dl, SCr 1.0 mg/dl, digoxin 1.5 ng/ml. His BP is 140/88 mmHg and pulse is 90/min.

Regarding LK’s HF management, which of the following describes the best management approach to take at this time, as it relates to his current medications?
a. Increase the furosemide dose for better BP and symptom control, and cut the digoxin dose in half.
b. Increase the lisinopril dose for better BP and symptom control, and cut the digoxin dose in half.
c. Increase the doses of lisinopril and furosemide for better BP and symptom control.
d. Increase the lisinopril dose for better BP and symptom control, cut the digoxin dose in half, and D/C the atorvastatin because of it’s ability to cause a worsening of HF symptoms.
b. Increase the lisinopril dose for better BP and symptom control, and cut the digoxin dose in half.

a and c are incorrect because you should not increase the furosemide dose unless the patient has congestion symptoms on the current dose. b is correct as this dose isn’t optimal and it should be increased to determine whether this will provide the needed BP control; it will likely also help control symptoms and reduce hospitalizations. Additionally, the serum digoxin concentration is too high – it should not exceed 1 and so the dose should be reduced to get it to this level. d is incorrect because atorvastatin does not have any effects of worsening HF symptoms.
Addition of which of the following drugs would NOT be a cause for concern (assuming it’s indicated), in a heart failure patient?
a. amoxicillin
b. naproxen
c. valdecoxib
d. propafenone
a. amoxicillin

Naproxen and valdecoxib have the potential to cause sodium and water retention (and thus worsen symptoms) and propafenone has negative inotropic effects.
TJ is a 67 year old African American male with a history of nonischemic cardiomyopathy (EF = 30%), and longstanding hypertension. His current medication regimen is:
lisinopril 20 mg bid, carvedilol 12.5 mg bid, furosemide 40 mg qd, KCl 20 mEq qd. His heart failure symptoms are currently well-controlled and he has minimal limitations to his activities.

VS: BP-148/72 mmHg
P-72/min R-12/min, afebrile

TJ is in for a 6 month follow-up visit. Which of the following would be most appropriate regarding TJ’s therapy?
a. Increase carvedilol to 25 mg bid
b. Add amlodipine 5 mg qd
c. Make no changes since his heart failure is currently well controlled
d. Add spironolactone 25 mg qd
a. Increase carvedilol to 25 mg bid

a is correct because the patient is not currently at the target carvedilol dose and his HR of 72 suggests he is not fully beta-blocked. Additionally his BP is poorly controlled and so an increase in carvedilol is appropriate. b is incorrect because you want to optimize standard HF drugs before adding others for BP control. c is incorrect because the BP is poorly controlled and BB dose not optimized for HF benefits. d is incorrect because the patient doesn’t meet the severity criterion. Additionally there is not sufficient information on the patient’s renal function or K to assess appropriateness of such an addition.
PL is a 45 year old African American male with difficult to control hypertension. Prior to treatment, his blood pressure was 166/110 mmHg. He is currently taking diltiazem SR 180 mg bid, HCTZ 25 mg qd and quinapril 40 mg qd. His blood pressure is typically 135-150/85-95 mmHg on this regimen.

Which of the following OTC medications would most likely adversely affect PL’s hypertension control and thus should be avoided?
a. pseudoephedrine
b. chlorpheniramine
c. loratidine
d. acetaminophen
a is correct because pseudoephedrine in a sympathomimetic with the known potential to raise BP. Chlorpheniramine and loratidine are H2-blockers, which do not raise BP. Similarly acetominophen does not raise BP.
PL is a 45 year old African American male with difficult to control hypertension. Prior to treatment, his blood pressure was 166/110 mmHg. He is currently taking diltiazem SR 180 mg bid, HCTZ 25 mg qd and quinapril 40 mg qd. His blood pressure is typically 135-150/85-95 mmHg on this regimen.

You have gone to a number of lengths to document that PL is adherent to therapy, yet most of his blood pressures are still slightly > 140/90 mmHg and so there is a desire for better blood pressure control. What would be the most appropriate recommendation regarding achieving better blood pressure control in PL?
a. The patient has achieved a substantial reduction in BP, so it is not necessary to produce a further lowering in BP.
b. Add eplerenone
c. Increase the diltiazem dose
d. Increase the HCTZ dose
c. Increase the diltiazem dose

a is incorrect because the patient has not achieved the goal BP, despite having a significant BP reduction. Thus additional lowering is warranted. b is incorrect because Eplerenone would not be considered until the current drug doses are optimized. c is correct because the patient has room to increase the diltiazem dose and CCBs are a group that due to kinetic variability should definitely be dose optimized. d is incorrect because doses above 25 mg typically provide little additional antihypertensive effect and many additional increases in risk of metabolic abnormalities, e.g. hypokalemia.
TT is a 76 year old African American female with a history of bradycardia, probably secondary to sick sinus syndrome. At present, her blood pressure is 172/82 mmHg and heart rate is 53/min. Her only cardiovascular medication is HCTZ 25 mg qd and she has no other significant cardiovascular disease history. Which of the following would be most appropriate for this patient?

a. Add metoprolol 50 mg twice daily
b. Add doxazosin 5 mg daily
c. Add amlodipine 5 mg daily
d. Optimize HCTZ dose to 50 mg daily
c. Add amlodipine 5 mg daily

a is incorrect because of the negative chronotropic effects. b is incorrect because alpha blockers are not considered first line therapy and additionally should not be considered early in therapy for elderly because of their potential to cause orthostatic hypotension. c is correct as this would be a safe drug to add and is likely to produce additional BP reduction. d is incorrect because generally you want to avoid doses of HCTZ above 25 mg since higher doses have minimal antihypertensive effect and more metabolic adverse effects.
OJ in a 54 year old African Ameraican female who was diagnosed with hypertension 3 months ago. She was started on diltiazem with a couple dose titrations, and has been taking diltiazem SR 360 mg daily for the last month. Her blood pressure has gone from 152/99 mmHg at baseline to 146/96 mmHg on the current dose. Which of the following would be the most appropriate recommendation for the management of OJ at this time?
a. add trandolapril
b. discontinue diltiazem and start chlorthalidone
c. add HCTZ
d. discontinue diltiazem and start metoprolol
d. discontinue diltiazem and start metoprolol

a is not the best option because it would mean adding a drug to one that is minimally effective (having produced a 4% reduction in SBP and 3% for DBP). b is not the most appropriate because the minimal response to diltiazem is likely to also translate into a minimal response to chlorthalidone. c is not most appropriate for several reasons; first adding to an ineffective drug doesn’t make sense, secondly the combination of a thiazide/CCB is pharmacologically not very logical. d is the best choice since this is a drug that works via the alternative pathophysiological mechanisms. Given the poor response to CCB, selection of an ACEI, ARB or BB as new monotherapy would be most logical.
WF is a 76 year old who was diagnosed with hypertension over a decade ago. She is currently taking HCTZ 25 mg daily and her BP is 166/84 mmHg. She has serious financial constraints and so drug cost is an important issue. Which of the following would be the most appropriate recommendation for WF?
a. Add nifedipine SR
b. Add candesartan
c. Add carvedilol
d. Add enalapril
d. Add enalapril

a is not the most appropriate choice because the combination of a CCB and thiazide is not an optimal combination. b is incorrect because it is a branded product and so will be quite expensive for the patient. c is incorrect because it is also a branded product and is very expensive. d is a good choice since it represents a logical combination and is available generically.
VM is a 52 year old Hispanic female with type 2 diabetes and hypertension. She is currently treated with ramipril 10 mg daily and her BP is 138/84 mmHg. Which of the following is the most appropriate recommendation?
a. continue current therapy as her BP is well controlled
b. add HCTZ
c. add verapamil
d. add metoprolol
c. add verapamil

a is incorrect because her BP is not to target for a diabetic. b is incorrect because it has metabolic adverse effects and agents without such effects should be attempted first. c is the best choice because it is a logical combination with an ACEI and also because it does not have any adverse effects in diabetics. d is incorrect because it is not a logical combination with ACEI and also because it has numerous potential problems in diabetics.
BP is a 37 year old white female with a 5 year history of hypertension, currently treated and controlled with enalapril. She comes to your pharmacy to purchase prenatal vitamins, and upon questioning, she tells you she is pregnant. Which of the following would be the most appropriate course of action that would likely lead to the best outcomes?
a. Encourage her to talk to her Ob/Gyn physician before the end of the first trimester to discuss changing her antihypertensive drug.
b. Contact her MD immediately and suggest changing her blood pressure medication to metoprolol
c. Contact her MD immediately and suggest changing her blood pressure medication to losartan
d. Contact her MD immediately and suggest changing her blood pressure medication to diltiazem
b. Contact her MD immediately and suggest changing her blood pressure medication to metoprolol

a is incorrect because recent evidence suggests that there are also problems with ACEI in the first trimester. b is the best choice because it is commonly used in pregnancy and works by generally the same pathophysiological mechanism as the ACEI, so is most likely to control her HTN. c is incorrect because ARBs are also contraindicated in pregnancy. d is not optimal because it works on the opposite pathophysiology as the ACEI. Thus, while it might be fine to use in pregnancy, is probably less likely than metoprolol to control her HTN.
JL is a 65 year old white female recently diagnosed with stage I hypertension. Her blood pressure at diagnosis averaged 148-156/94-98 mmHg. She was started on atenolol 50 mg daily, and then increased two weeks ago to 100 mg daily for inadequate blood pressure control on the initial dose. She has call your pharmacy because she has felt extremely fatigued since the most recent dose increase. She takes her blood pressure at home and reports that this AM it was: BP 134/86 mmHg, HR 50/min

What would be the most appropriate recommendation for management of JL?

a. discontinue atenolol and start enalapril
b. discontinue atenolol and start felodipine
c. reduce atenolol dose to 50 mg daily and add chlorthalidone
d. consider switching from atenolol to metoprolol since she may be sleeping poorly (leading to the fatigue) and this would likely improve with metoprolol
a. discontinue atenolol and start enalapril

a is the best choice because she had a good BP response to atenolol but at the dose with a good response she had symptomatic bradycardia. b would be less optimal because it switches treatment approaches to the other mechanism. c is not logical because the lower dose of atenolol did not control her BP and this approach would lead to use of 2 drugs when 1 may suffice. d is not correct because the fatigue she is experiencing is due to bradycardia and reports of dreams/nightmares with beta-blockers are not thought to differ substantially between drugs in the class.
KW is a 55 year old woman with a known history of asthma who presents to the emergency department with substernal chest pain of unknown duration. Initial work-up reveals the following: VSs – BP 120/80 mmHg, HR 88, RR 22, T afebrile. ECG: non-specific ST changes. Initial impression is unstable angina and the patient is prepared for cardiac catheterization. The following two questions refer to this patient. Which of the following would be LEAST beneficial initial therapy for this patient?
a. Metoprolol (Lopressor)
b. Verapamil (Verelan and others)
c. Sublingual nitroglycerin
d. Diltiazem (Cardizem, Dilacor, others)
a. Metoprolol (Lopressor)

Selection “a” is correct; please see objectives 2 and 5. Metoprolol is contraindicated in this patient since she has a history of asthma.
OH is a 61 year old former professional baseball player who complains of a sudden onset of chest pain and shortness of breath beginning approximately 4 hours ago. Initial evaluation reveals the following: Vital Signs – BP 90/60 mmHg, HR 72, RR 22, T afebrile. ECG – ST depression in leads II, III, AVF. Labs – troponin positive. His physical exam is notable for pulmonary rales, +JVD, and peripheral edema. Initial impression is a non-ST segment elevation myocardial infarction with moderate to severe heart failure. The next two questions refer to this patient. Which of the following correctly characterizes use of reteplase (Retevase) in this patient?

a. Unfractionated heparin should also be used to prevent re-occlusion.
b. Enoxaparin (Lovenox) is preferred over unfractionated heparin for prevention of re-occlusion.
c. Reteplase (Retevase) may be administered without either enoxaparin or unfractionated heparin
d. Reteplase (Retevase) is not indicated for this patient.
d. Reteplase (Retevase) is not indicated for this patient.

Selection “d” is correct; please see objectives 2 and 5. Reteplase is a thrombolytic and indicated only for management of STEMI. This patient has NSTEMI.
OH is a 61 year old former professional baseball player who complains of a sudden onset of chest pain and shortness of breath beginning approximately 4 hours ago. Initial evaluation reveals the following: Vital Signs – BP 90/60 mmHg, HR 72, RR 22, T afebrile. ECG – ST depression in leads II, III, AVF. Labs – troponin positive. His physical exam is notable for pulmonary rales, +JVD, and peripheral edema. Initial impression is a non-ST segment elevation myocardial infarction with moderate to severe heart failure. The next two questions refer to this patient. Which of the following correctly characterizes use of reteplase (Retevase) in this patient?

Which of the following would be LEAST beneficial for this patient?

a. Aspirin
b. Metoprolol (Lopressor)
c. Eptifibatide (Integrilin)
d. Intravenous nitroglycerin
b. Metoprolol (Lopressor)

Selection “b” is correct; please see objectives 2 and 5. Since patient has moderate to severe heart failure, the beta blocker metoprolol may make this condition worse. Intravenous NTG may also exacerbate OH’s hypotension but, if judiciously administered, may also relieve the pulmonary congestion and peripheral hypoperfusion. ASA and eptifibatide are the most beneficial agents on this list (see slide #37).
BS is a 45 year old woman who has suffered an ST segment elevation myocardial (STEMI) infarction and is about to receive tissue plasminogen activator. Which of the following correctly describe beneficial effects of this drug on the pathophysiology of STEMI?

a. Normalizes endothelial dysfunction
b. Increases myocardial oxygen demand
c. Increases myocardial oxygen supply
d. Increases platelet activation
c. Increases myocardial oxygen supply

Selection “c” is correct; please see objectives 1 and 3. A fibrinolytic such as tissue plasminogen activator will, in 55% to 75% of patients, lyse an intra-arterial clot and re-establish normal/near normal coronary artery blood flow. This change in flow increases myocardial oxygen supply.
KK is a 45 year old man who presents to his local emergency room with sub-sternal chest pain and is subsequently found to have an ST segment elevation myocardial infarction. Therapy with tenecteplase (TNKase) is planned.

Which of the following would be an absolute contraindication to use of this agent in this patient?

a. Blood pressure >180/110 mmHg
b. History of a gastrointestinal bleed 3 weeks ago
c. History of a cerebral hemorrhage 2 years ago
d. History of heparin-induced thrombocytopenia
c. History of a cerebral hemorrhage 2 years ago

Selection “c” is correct; please see objectives 2 and 5. Any cerebral hemorrhage represents an absolute contraindication to any thrombolytic agent regardless of how long ago it occurred. Please see slide #49.
KK is a 45 year old man who presents to his local emergency room with sub-sternal chest pain and is subsequently found to have an ST segment elevation myocardial infarction. Therapy with tenecteplase (TNKase) is planned.

Which of the following describes appropriate anticoagulation for this patient?

a. Use only if he has a high risk of systemic embolism
b. Give unfractionated heparin 5000 units subcutaneously bid
c. Systemic anticoagulation is unnecessary
d. Use of enoxaparin would be permissible in this patient
d. Use of enoxaparin would be permissible in this patient

Selection “d” is correct; please see objectives 2 and 5. Selection “a” and “c” describes appropriate use of anticoagulation if streptokinase were used. The dose of unfractionated heparin is selection “b” is appropriate only for prevention of a venous thromboembolism. Please see slide #50.
Upon discharge from the hospital, which of the following is LEAST likely to prevent a subsequent acute coronary event?

a. Aspirin
b. Isosorbide mononitrate (Imdur and others)
c. Aspirin + Clopidogrel (Plavix)
d. Pravastatin (Pravachol)
b. Isosorbide mononitrate (Imdur and others)
LK is a 52-year-old African-American male with a history of hyperlipidemia, type 2 diabetes mellitus, HTN and heart failure. His heart failure symptoms are currently classified as Class III. His current medications include atorvastatin (Lipitor) 80 mg po qd, nateglinide (Starlix) 120 mg tid, acarbose (Precose) 50 mg tid, lisinopril (Zestril, Prinivil) 10 mg qd, furosemide (Lasix) 60 mg qd, digoxin (Lanoxin) 0.25 mg qd. Current labs are as follows: FBG 130 mg/dl, HbA1c 7.6%, Na – 135 mEq/L, K – 4.3 mEq/L, CL – 99 mEq/L, BUN 16 mg/dl, SCr 1.0 mg/dl, digoxin 1.5 ng/ml. His BP is 140/88 and pulse is 90.
The physician raises the issue of starting a B-blocker in this patient. Which of the following is the most appropriate response?

a. While B-blockers are beneficial in heart failure, it would be contraindicated in this patient because of his diabetes.
b. B-blocker is strongly indicated, and therapy should be initiated with carvedilol 12.5 mg bid.
c. B-blocker is strongly indicated, and therapy should be initiated with Toprol XL 12.5 mg qd
d. B-blocker should be started, but discontinued if the patient has any adverse changes in their lipid parameters.
c. B-blocker is strongly indicated, and therapy should be initiated with Toprol XL 12.5 mg qd
LK is a 52-year-old African-American male with a history of hyperlipidemia, type 2 diabetes mellitus, HTN and heart failure. His heart failure symptoms are currently classified as Class III. His current medications include atorvastatin (Lipitor) 80 mg po qd, nateglinide (Starlix) 120 mg tid, acarbose (Precose) 50 mg tid, lisinopril (Zestril, Prinivil) 10 mg qd, furosemide (Lasix) 60 mg qd, digoxin (Lanoxin) 0.25 mg qd. Current labs are as follows: FBG 130 mg/dl, HbA1c 7.6%, Na – 135 mEq/L, K – 4.3 mEq/L, CL – 99 mEq/L, BUN 16 mg/dl, SCr 1.0 mg/dl, digoxin 1.5 ng/ml. His BP is 140/88 and pulse is 90.

Regarding LK’s current HF medications, which of the following best describes the management approach you would recommend?

a. Increase the furosemide (Lasix) dose for better BP and symptom control, and cut the digoxin dose in half.
b. Increase the lisinopril (Zestril, Prinivil) dose for better BP and symptom control, and cut the digoxin (Lanoxin) dose in half.
c. Increase the doses of lisinopril (Zestril, Prinivil) and furosemide (Lasix) for better BP and symptom control.
d. Increase the lisinopril (Zestril, Prinivil) dose for better BP and symptom control, cut the digoxin (Lanoxin) dose in half, and D/C the atorvastatin (Lipitor) because of it’s ability to cause a worsening of HF symptoms.
b. Increase the lisinopril (Zestril, Prinivil) dose for better BP and symptom control, and cut the digoxin (Lanoxin) dose in half.
TJ is a 67 yo BM with a history of nonischemic cardiomyopathy (EF = 30%), and longstanding hypertension. His current medication regimen is: lisinopril (Zestril, Prinivil) 20 mg bid, carvedilol (Coreg) 12.5 mg bid, furosemide (Lasix) 40 mg qd, KCl (K-dur) 20 mEq qd. His heart failure symptoms are currently well-controlled and he has minimal limitations to his activities.
VS: BP – 148/72 mm Hg; P – 72 bpm, R – 12, afebrile

TJ is in for a 6 month follow-up visit. Which of the following would you recommend regarding TJ’s therapy?
a. Increase carvedilol to 25 mg bid
b. Add amlodipine 5 mg qd
c. Make no changes since his heart failure is currently well controlled
d. Add spironolactone 25 mg qd
a. Increase carvedilol to 25 mg bid
PL is a 45-year-old black male with difficult to control hypertension. Prior to treatment, his BP was 166/110 mmHg. He is currently taking diltiazem (Cardizem, Dilacor, others) SR 180 mg bid, hydrochlorothiazide (Esidrix and others) 25 mg qd and quinapril (Accupril) 40 mg qd. His blood pressure is typically 135-150/85-95 mm Hg on this regimen.

Which of the following medications might adversely affect PL’s hypertension control, and thus should be avoided?

a. pseudoephedrine (Sudafed, others)
b. diphenhydramine (Benadryl, others)
c. loratadine (Claritin)
d. acetaminophen (Tylenol)
a. pseudoephedrine (Sudafed, others)
PL is a 45-year-old black male with difficult to control hypertension. Prior to treatment, his BP was 166/110 mmHg. He is currently taking diltiazem (Cardizem, Dilacor, others) SR 180 mg bid, hydrochlorothiazide (Esidrix and others) 25 mg qd and quinapril (Accupril) 40 mg qd. His blood pressure is typically 135-150/85-95 mm Hg on this regimen.

You have gone to a number of lengths to document that PL is adherent to therapy, yet most of his blood pressures (BP) are still slightly > 140/90 and so there is a desire for better BP control. What would be the most appropriate recommendation regarding achieving better BP control in PL?

a. The patient has achieved a substantial reduction in BP, so it is not necessary to produce a further lowering in BP.
b. Add eplerenone (Inspra)
c. Increase the diltiazem (Cardizem, Dilacor, others) dose
d. Increase the hydrochlorothiazide dose
c. Increase the diltiazem (Cardizem, Dilacor, others) dose
WR is a 40 yo BF with no significant medical history except for a recent diagnosis of hypertension. Based on JNC 7, which of the following is preferred therapy for WR, and most likely to produce an adequate blood pressure reduction?

a. lisinopril (Zestril, Prinivil)
b. atenolol (Tenormin)
c. diltiazem (Cardizem, Dilacor, others)
d. chlorthalidone (Hygroton)
d. chlorthalidone (Hygroton)
RH is a 54 yo WM with a 10 year history of hypertension. His blood pressure is currently controlled with lisinopril (Zestril, Prinivil) 20 mg qd. He recently has developed symptoms of chest pain with exertion, and was diagnosed with chronic stable angina. He presently has anginal symptoms about 2-3 times per week. Among the following, which is the most rational approach to management of RH?

a. add sublingual nitroglycerin
b. add diltiazem (Cardizem, Dilacor, others) and sublingual nitroglycerin
c. add nifedipine (Procardia) SR and sublingual nitroglyercin and discontinue lisinopril (Zestril, Prinivil)
d. add metoprolol (Lopressor, Toprol) and sublingual nitroglycerin and discontinue lisinopril
b. add diltiazem (Cardizem, Dilacor, others) and sublingual nitroglycerin
TT is a 76 yo BF with a history of bradycardia, probably secondary to sick sinus syndrome. At present, her blood pressure is 172/82 and HR is 61 beats per minute. Her only cardiovascular medication is hydrochlorothiazide (Esidrix and others) 25 mg qd and she has no other significant cardiovascular disease history. Which of the following would you recommend?

a. No additional therapy since her blood pressure is well controlled on hydrochlorothizide
b. Add amlodipine (Norvasc) 5 mg qd
c. Add metoprolol (Toprol, Lopressor) 50 mg bid
d. Add doxazosin (Cardura) 5 mg qd
b. Add amlodipine (Norvasc) 5 mg qd
Refer to Case D

The medical term used for the patient description of “awaking during the night with a feeling of suffocating” is:

a. orthopnea
b. dyspnea
c. paroxysmal nocturnal dyspnea
d. increased preload
c. paroxysmal nocturnal dyspnea
Refer to Case D

Regarding the patient’s ACE inhibitor therapy, you are recommending a dosage increase. The justification for this increase is:

a. The patient is not receiving the dose documented in clinical trials to reduce mortality
b. The dose increase is needed in order for the patient to eliminate the 20 pounds of excess fluid she has accumulated in recent weeks
c. The patient is receiving a mortality-reducing dose, but further dose increase will improve her control of symptoms
d. The dose increase is needed to get her to a mortality-reducing dose, and to improve her symptomatic control.
d. The dose increase is needed to get her to a mortality-reducing dose, and to improve her symptomatic control.
Refer to Case D

Regarding the patient’s B-blocker therapy, which of the following is most accurate?

a. Her B-blocker dose should not be increased due to her vital signs, but since she is not at the target dose, she is unlikely to derive the expected benefits.
b. Her B-blocker should be discontinued because her pulse and BP are too low.
c. This patient’s B-blocker therapy would be considered to be optimized.
d. The patient would be expected to have a dramatic worsening of HF symptoms if her B-blocker was discontinued.
c. This patient’s B-blocker therapy would be considered to be optimized.
Refer to Case D

Which of the following is the most appropriate recommendation regarding addition of therapy in this patient?
a. Add KCl (K-dur) 40 mEq qd
b. Add spironolactone (Aldactone) 25 mg qd
c. Add candesartan (Atacand)
d. No additional therapy is recommended at this time.
b. Add spironolactone (Aldactone) 25 mg qd
LK is a 48 yo WM who was recently been diagnosed with HTN. He has been
taking lisinopril (Zestril, Prinivil) 40 mg qd for 2 months and his BP is currently 148/100 mmHg (untreated avg BP – 154/100). Which of the following would represent the most appropriate management of LK?
a. Continue the lisinopril (Zestril, Prinivil) for an additional month as it often takes some time before the BP control is evident.
b. Add HCTZ 12.5 mg to lisinopril (Zestril, Prinivil)
c. D/C lisinopril (Zestril, Prinivil) and start atenolol (Tenormin) 50 mg qd
d. D/C lisinopril (Zestril, Prinivil) and start diltiazem (Cardizem, Dilacor, others) SR 180 mg qd
d. D/C lisinopril (Zestril, Prinivil) and start diltiazem (Cardizem, Dilacor, others) SR 180 mg qd
KG is a 42 yo BF with a history of obesity and Type 2 DM, who was recently diagnosed with hypertension. Her BP is currently 162/100. Which of the following would be the most appropriate recommendation for initial management of KG’s HTN?
a. Candesartan (Atacand)
b. Trandolapril (Mavik) + verapamil (Verelan, others)
c. hydrochlorothiazide (Esidrix and others)
d. Lisinopril (Zestril, Prinivil) + HCTZ
b. Trandolapril (Mavik) + verapamil (Verelan, others)
Given the following EKG changes and presentations, which patient would derive the most
benefit from a fibrinolytic agent?
a. Televation inferior wall myocardial infarction who presents 12 hours after symptom
onset
b. ST elevation anterior wall myocardial infarction who presents within 4 hours of
symptom onset
c. New left bundle branch block presenting 18 hours after symptom onset
d. ST depression > 1 mm on 2 contiguous leads presenting within 1 hour of symptoms
b. ST elevation anterior wall myocardial infarction who presents within 4 hours of
symptom onset
Which of the following is correct regarding clopidogrel (Plavix) in acute coronary
syndromes?
a. Clopidogrel (Plavix) is recommended in conjunction with stenting in individuals
undergoing percutaneous coronary angioplasty in the setting of ST elevation myocardial
infarction. A loading dose of 300 mg should be given prior to the intervention, followed
by 75 mg daily for life.
b. Clopidogrel (Plavix) is recommended in the medical management of patients with NSTE acute coronary syndromes, especially in institutions where early cath lab-based
interventions are not routinely performed. In this scenario, patients should also receive
aspirin.
c. The benefit of clopidogrel (Plavix) in acute coronary syndromes is due to its ability to inhibit platelet activation. This results in a reduced quadruple endpoint of cardiovascular
death, myocardial infarction, stroke, and bleeding by 20% when used with aspirin.
d. When clopidogrel (Plavix) is used, patients should also receive concomitant glycoprotein
IIbIIIa inhibitors. The majority of the data support the use of this combination, especially
in early non-interventional strategies (CURE trial).
b. Clopidogrel (Plavix) is recommended in the medical management of patients with NSTE acute coronary syndromes, especially in institutions where early cath lab-based
interventions are not routinely performed. In this scenario, patients should also receive
aspirin.
A 65 year old woman with a history of stroke (6 months ago) presents to the emergency room
with Stelevation anterior wall myocardial infarction within 2 hours of symptom onset. Which
of the following is the best strategy for this patient?
a. Aspirin, heparin, iv nitroglycerin, reteplase (Retevase), b-blocker
b. Aspirin, heparin, iv nitroglycerin, tenecteplase (TNKase), b-blocker, ACE inhibitor
c. Aspirin, heparin, iv nitroglycerin, angioplasty, b-blocker, ACE inhibitor
d. Aspirin, enoxaparin (Lovenox), tenecteplase (TNKase), iv nitroglycerin, b-blocker, ACE inhibitor
c. Aspirin, heparin, iv nitroglycerin, angioplasty, b-blocker, ACE inhibitor
Which of the following GPIIbIIIa inhibitors should not be used in medical management (noninterventional)
of high risk non ST elevation acute coronary syndrome patients?
a. Eptifibatide (Integrilin)
b. Abciximab (ReoPro)
c. Tirofiban (Aggrastat)
d. GPIIbIIIa inhibitors should not be used at all in these patients
b. Abciximab (ReoPro)
Which of the following statements is true regarding enoxaparin (Lovenox)?
a. Enoxaparin (Lovenox) has been studied in combination with fibrinolytics in ST elevation
myocardial infarction. This combination has resulted in decreased ischemic events and a
lower incidence of bleeding when compared with fibrinolytic/heparin combination.
b. Enoxaparin (Lovenox) has been shown to be superior to heparin in the setting of non ST
elevation myocardial infarction and unstable angina. However, anticoagulation with
enoxaparin (Lovenox) cannot be monitored and should therefore not be used in patients
with renal dysfunction.
c. Enoxaparin (Lovenox) has more predictable pharmacokinetics than unfractionated
heparin. In addition, it is administered subcutaneously, and reduces the rates of
revascularization in non ST elevation acute coronary syndrome when compared with
unfractionated heparin.
d. Enoxaparin (Lovenox) has no benefit in hospitals that do not have high rates of cath labbased reperfusion procedures. It should not be used in conjunction with GP IIbIIIa
inhibitors. If using enoxaparin, anti-Xa levels should be routinely monitored.
c. Enoxaparin (Lovenox) has more predictable pharmacokinetics than unfractionated
heparin. In addition, it is administered subcutaneously, and reduces the rates of
revascularization in non ST elevation acute coronary syndrome when compared with
unfractionated heparin.
Refer to Case E

Shortness of breath (SOB) is primarily the consequence of activation of which of the
following compensatory mechanisms in heart failure?
a. Increased preload
b. Vasoconstriction
c. Sympathetic nervous system activation
d. Left ventricular hypertrophy
a. Increased preload
Refer to Case E

Regarding this patient’s digoxin (Lanoxin) therapy, which of the following is most
appropriate?
a. Decrease dose to 0.125 mg/d as the patient has classic signs and symptoms of digoxin
(Lanoxin) toxicity.
b. Decrease dose to 0.125 mg/d since data suggest that digoxin (Lanoxin) concentrations in
the range of the patient’s are associated with increased risk of adverse outcomes,
including death.
c. Discontinue digoxin (Lanoxin) as the patient is not a good candidate for this therapy and
is unlikely to derive any benefit.
d. No changes in the digoxin (Lanoxin) therapy are warranted at present.
b. Decrease dose to 0.125 mg/d since data suggest that digoxin (Lanoxin) concentrations in
the range of the patient’s are associated with increased risk of adverse outcomes,
including death.
Refer to Case E

Which of the following would be the most appropriate recommendation to improve the
symptomatic management of this patient?
a. Increase lisinopril (Zestril, Prinivil) dose to 20 mg qd
b. Increase Toprol XL (metoprololo) dose to 50 mg qd
c. Increase both lisinopril (Zestril, Prinivil) and Toprol XL (metoprolol) doses
d. Increase furosemide (Lasix) to 120 mg qd for the next three weeks
a. Increase lisinopril (Zestril, Prinivil) dose to 20 mg qd
Refer to Case E

The above recommendation is accepted and doses are appropriately titrated. At the patient’s
clinic visit one month later, he complains of no significant symptoms and his vitals are: BP:
146/88 mmHg; HR: 58 bpm, RR: 12 rpm, afebrile. Which of the following would be most
appropriate?
a. Recommend no changes in therapy at this time
b. Add amlodipine (Norvasc) 5 mg qd
c. Add candesartan (Atacand) 4 mg qd
d. Discontinue esomeprazole (Nexium)
c. Add candesartan (Atacand) 4 mg qd
Refer to Case E

RT’s insurance provider has called about making a formulary switch of his Toprol XL to
generic, immediate release metoprolol. Which of the following would be the most
appropriate response to this request?
a. This would be an appropriate switch, since it’s the same drug.
b. Immediate release metoprolol (Lopressor) has not been studied in heart failure so it would be unclear what dose would be most appropriate.
c. Immediate release metoprolol (Lopressor) is the best studied of the b-blockers, so this is not only reasonable from a cost perspective, but represents the practice of evidence-based medicine.
d. This switch would be unadvisable since immediate release metoprolol (Lopressor) was inferior in a b-blocker comparator trial.
d. This switch would be unadvisable since immediate release metoprolol (Lopressor) was inferior in a b-blocker comparator trial.
RT is a 74 year old white female, recently diagnosed with systolic heart failure. The
presumed etiology of her heart failure is hypertension and her medical history is otherwise
relatively insignificant. The patient has minimal shortness of breath (SOB) and dyspnea on exertion (DOE) and trace pedal edema. She complains of a dry, tickly cough that has been present since about the time she was diagnosed with heart failure, and she finds it moderately bothersome.
Current medications (unchanged for the past 3 months): enalapril (Vasotec) 10 mg bid, carvedilol (Coreg) 12.5 mg bid, furosemide (Lasix) 20 mg qd.
VS: BP – 106/52 P – 56 R – 14

Which of the following would be most appropriate?
a. Reduce carvedilol (Coreg) dose
b. Change enalapril (Vasotec) to ramipril (Altace) since it’s more tissue specific
c. D/C enalapril (Vasotec) and start candesartan (Atacand)
d. D/C enalapril (Vasotec) and start hydralazine-nitrate combination
c. D/C enalapril (Vasotec) and start candesartan (Atacand)
DF is a 71 year old white female with a 3 year history of heart failure secondary to ischemic heart disease. She was recently hospitalized for an exacerbation of heart failure.
Current medications:
Ramipril (Altace) 5 mg bid
Carvedilol (Coreg) 12.5 mg bid
Furosemide (Lasix) 20 mg qd
Digoxin (Lanoxin) 0.125 mg qd
ASA 81 mg qd
Lovastatin (Mevacor) 40 mg qhs
VS: P - 78 beats/min, BP - 115/64 mmHg, R – 19/min, T - 98.6°F.
Labs: Na – 132 mEq/L, K – 4.7 mEq/L; Cl – 98 mEq/L; Bicarbonate – 26 mEq/L;
BUN – 28 mg/dl; SCr – 2.6 mg/dl; glucose – 79 mg/dl
Which of the following would be most appropriate for this patient?
a. Increase carvedilol (Coreg) to 25 mg bid
b. Add spironolactone (Aldactone) 25 mg qd
c. Increase ramipril (Altace) to 10 mg bid
d. Increase carvedilol (Coreg) to 25 mg bid and add spironolactone (Aldactone)
a. Increase carvedilol (Coreg) to 25 mg bid
HL is a 54 year old bla ck male with chronic renal insufficiency (creatinine clearance < 40 ml/min). His HTN is currently treated with enalapril (Vasotec) 20 mg bid and his blood pressure is 136/88 mm Hg. Last year’s exam asked whether the most appropriate
recommendation was to: 1) Add HCTZ 25 mg qd; 2) Add furosemide 20 mg qd; 3) Add
losartan 50 mg qd or 4) No change in therapy is warranted as the patient is on an appropriate drug regimen that has brought him to his BP goal. The option to “add furosemide 20 mg qd”
was correct. The reason this was the correct answer is:
a. Loop diuretics are much more potent antihypertensive drugs than thiazides.
b. Loop diuretics have better data with regards to reducing negative outcomes associated
with hypertension.
c. Loop diuretics produce more potassium wasting, which is desirable in a patient with renal insufficiency.
d. Loop diuretics maintain efficacy in patients with renal impairment whereas thiazides do not.
d. Loop diuretics maintain efficacy in patients with renal impairment whereas thiazides do not.
PR is 45 year old white male who comes to your pharmacy. He was recently diagnosed with
hypertension and is getting his first antihypertensive prescription filled. He tells you that his
blood pressures over the last month have average 156/98 mmHg. He has no other significant
past medical history. Based on JNC 7 guidelines, which of the following would be the most
appropriate initial antihypertensive therapy for PR?
a. Atenolol (Tenormin)
b. Trandolapril (Mavik)
c. Chlorthalidone (Hygroton)
d. Diltiazem (Cardizem, Dilacor and others)
c. Chlorthalidone (Hygroton)
WW is a 46 year old black male who takes his blood pressure in your pharmacy. Today’s reading is 134/82 mmHg, and this is fairly consistent with what it has been recently. WW has no significant past medical history. Which of the following is the most appropriate
advice/counseling for WW?
a. This is a normal blood pressure, and he should just make sure he checks it at least once per year.
b. This is considered prehypertension and initiation of lifestyle modifications is
recommended.
c. This is considered prehypertension and initiation of low-dose diuretic is warranted to prevent further elevations in blood pressure and a diagnosis of hypertension.
d. If this blood pressure is confirmed at a subsequent time, it would represent hypertension and would warrant initiation of drug therapy.
b. This is considered prehypertension and initiation of lifestyle modifications is
recommended.
KR is a 74 year old white female whose blood pressures over the last month have been
documented to be in the range of 162-170/76-84 mmHg. Which of he following is most
appropriate?
a. Initiate HCTZ .
b. Initiate diltiazem (Cardizem, Dilacor and others).
c. Initiate HCTZ (Hydrodiuril) + amlodipine (Norvasc).
d. Initiate HCTZ (Hydrodiuril) + labetalol (Normodyne, Trandate).
e. Initiate lifestyle modifications, but drug therapy is not indicated.
c. Initiate HCTZ (Hydrodiuril) + amlodipine (Norvasc).
LL is a 56 year old black male with a 10 year history of hypertension, currently treated and
controlled with chlorthalidone (Hygroton). He was admitted last evening for acute coronary
syndrome and has now been diagnosed with an anterior ST elevation myocardial infarction.
Which of the following is the most appropriate course for management of this patient’s
hypertension?
a. Continue chlorthalidone monotherapy
b. Add metoprolol (Lopressor, Toprol XL)
c. Add metoprolol (Lopressor, Toprol XL) and enalapril (Vasotec)
d. D/C chlorthalidone and start metoprolol and enalapril
d. D/C chlorthalidone and start metoprolol and enalapril
BM is a 42 year old black female with a history of asthma since childhood, and for which she
is currently taking inhaled corticosteroids and prn b-agonists. She was recently diagnosed
with hypertension and was started on HCTZ (Hydrodiuril) 25 mg qd. This has provided little
to no blood pressure lowering. Which of the following would be the most appropriate
recommendation regarding BM’s therapy?
a. Add diltiazem (Cardizem, Dilacor and others).
b. D/C HCTZ (Hydrodiuril) and start amlodipine (Norvasc).
c. D/C HCTZ (Hydrodiuril) and start quinapril (Accupril).
d. Discontine HCTZ (Hydrodiuril) and start metoprolol (Lopressor)
b. D/C HCTZ (Hydrodiuril) and start amlodipine (Norvasc).
RT is a 64 year old Korean American male with a history of hypertension, chronic stable
angina and hyperlipidemia. His current drug regimen is atenolol (Tenormin) 50 mg qd, ASA
81 mg qd, atorvastatin (Lipitor) 20 mg qHS and SL nitroglycerin (NTG) PRN. He reports
use of 2 SL NTG tablets over the previous two months. Vitals at present are: 148/82 mmHg,
P- 61 beats/min, RR –13/min. Which of the following is most appropriate?
a. No changes in his drug therapy are needed at present.
b. Add amlodipine (Norvasc).
c. Add ramipril (Altace).
d. Increase atenolol (Tenormin) to 100 mg qd.
c. Add ramipril (Altace).
TH is a 56 year old white female with a history of hypertension, dyslipidemia and Type 2
DM. Her hypertension is currently treated with lisinopril (Zestril, Prinivil) and her current
BP is 136/86. Which of the following would be the most appropriate recommendation
regarding TH’s hypertension management?
a. Add HCTZ (Hydrodiuril).
b. Add atenolol (Tenormin).
c. Add losartan (Cozaar).
d. Add diltiazem (Cardizem, Dilacor, others).
e. No changes in TH’s therapy are recommended.
d. Add diltiazem (Cardizem, Dilacor, others).
Refer to Case F

When given the question of which of the following would be the most appropriate
recommendation to improve the symptomatic management of this patient? 1) Increase
lisinopril dose to 20 mg qd; or 2) Increase Toprol XL dose to 50 mg qd; or 3) Increase
both lisinopril and Toprol XL doses; or 4) Increase furosemide to 120 mg qd for three weeks,
choice "1" (Increase lisinopril dose to 20 mg qd) was the correct answer. Which of the following correctly explains why one of the alternate choices was incorrect?
A. Furosemide is not useful for symptomatic management in a patient with an acute
decompensation
B. Furosemide's primary benefits are with symptomatic management of patients, but the benefits are not seen immediately
C. A three-fold increase in furosemide dose is a too excessive dose increase to make immediately
D. Toprol XL is used for its symptomatic benefits but the patient's heart rate is too low for a further dose increase.
C. A three-fold increase in furosemide dose is a too excessive dose increase to make immediately

A is false - is useful; B is false - effects seen immediately; D is false -
while HR is too low, β-blockers are not used in HF because of their symptomatic benefit
BM is a 42 year-old black female with a history of asthma since childhood, and for which she is currently taking inhaled corticosteroids and prn β-agonists. She was recently diagnosed
with hypertension, and was started on hydrochlorothiazide (HCTZ) 25 mg qd. This has
provided little to no BP lowering.
Given the choices of: 1) add diltiazem (Cardizem); 2) D/C HCTZ and start amlodipine
(Norvasc); 3) D/C HCTZ and start quinapril (Accupril); and 4) Increase HCTZ to 50 mg qd,
choice "2" (D/C HCTZ and start amlodipine) is obviously the best answer. Which of
the following best describes why choice "2" is correct, or the others were less appropriate?
A. HCTZ and diltiazem both cause hypokalemia, so the two of them should not be given together
B. Amlodipine is substantially better than other first-line antihypertensives at lowering BP, so would be most likely to get this patient to her BP goal.
C. Doses of HCTZ 50 mg qd usually result in significantly more BP lowering than 25 mg qd, but only if the patient has had some BP lowering at 25 mg qd
D. IF the quinapril caused a cough in this patient, it could potentially worsen her
asthma.
D. IF the quinapril caused a cough in this patient, it could potentially worsen her
asthma.

a) false because dilt does not cause hypokalemia; b) fasle because
amlodipine is not substantially better than others from a populatin perspective; c) false
because 50 mg doses provide little additional BP lowering
HL is a 54 year old black male with chronic renal insufficiency (creatinine clearance = 28 ml/min) with proteinuria. He’s been hypertensive for years, but hasn’t been treated. His current blood pressure is 156/98 mmHg. Which of the following is the most appropriate recommendation regarding his hypertension therapy?
a. Initiate enalapril
b. Initiate amlodipine + furosemide
c. Initiate irbesartan + chlorthalidone
d. Initiate ramipril + torsemide
e. Initiate amlodipine + lisinopril
d. Initiate ramipril + torsemide

This patient has a BP goal of < 130/80 (actually a lower BP would be better given proteinuria) and is highly unlikely to achieve this goal with monotherapy (a incorrect). All patients with renal disease should be on an ACE inhibitor or ARB (ACEI preferred) (thus B incorrect), and due to the fluid retention associated with renal disease, and diuretic is the most logical additional therapy. However, given this patient’s renal function, a thiazide would be ineffective (c incorrect). While a CCB with ACEI could be considered, most renal failure patients will require diuretic to control their HTN, so (d) preferred over (e).
RE has hypertension. Which of the following over-the-counter medications or products would you encourage her to avoid?
1) ketoprofen 2) loratadine 3) dextromethorphan 4) pseudoephedrine 5) ranitidine 6) ephedra weight loss products 7) omeprazole 8) chlorpheniramine
a. 1, 4, 6
b. 1, 3, 6
c. 2, 4, 8
d. 1, 7, 8
e. 1, 4, 5
a. 1, 4, 6
OTC products to avoid include NSAIDS (ketoprofen), and sympathomimetics (e.g. decongestants (pseudoephrine) and ephedra-containing products). Antihistaimes (e.g. loratidine, chlorpheniramine), cough suppressants (dextromethorphan), H2-blockers (ranitidine) or proton pump inhibitors (omeprazole) are not problematic in hypertensive patients.
Refer to Case F
Consideration is being given to starting this patient on spironolactone. Which of the following is the most appropriate response to this suggestion?
a. Spironolactone (and eplerenone) should be avoided in this patient
b. Given patient’s class III heart failure status and recent exacerbation, spironolactone is indicated and should be started
c. Eplerenone should be recommended since spironolactone could be problematic in this patient
d. This patient has no indication for spironolactone
e. Spironolactone has only been studied in African Americans so its benefits in this patient are unclear, thus should probably be avoided.
a. Spironolactone (and eplerenone) should be avoided in this patient

While this patient is a good candidate in general for an aldosterone antagonist, they should be avoided in him, due to his poor renal function and high K. It is not true that spironolactone has only been studied in African Americans, and if this patient had low K and good renal function, he might be a reasonable candidate for such therapy.
A 49 year-old man with anterior wall ST-segment elevated myocardial infarction is sent to the coronary catheterization lab for primary angioplasty. His course is uncomplicated and revascularization with stent placement is successful. He had no history of coronary disease before this event. His medical history is notable for childhood asthma, migraine headaches, and generalized anxiety disorder. His allergies include peanuts, penicillin, and aspirin. You are asked to write orders for his discharge medications by the cardiology fellow. Which of the following is the BEST regimen for this patient?
a. Metoprolol, fosinopril, aspirin, clopidogrel, and SL nitroglycerin
b. Atenolol, ramipril, clopidogrel, and isosorbide dinitrate prn
c. Atenolol, captopril, simvastatin, clopidogrel, and SL nitroglycerin
d. Lisinopril, clopidogrel, aspirin, diltiazem, and SL nitroglycerin
c. Atenolol, captopril, simvastatin, clopidogrel, and SL nitroglycerin

Correct answer is C. B is incorrect because isosorbide dinitrate is not a prn drug. D
is incorrect because a beta-blocker (not diltiazem) should be given. C is a superior
choice to A because a statin is part of the treatment plan. A and D are also incorrect
since they include ASA, a medication to which the patient is allergic.
FJ is a 64 year old man with a long (many years) history of chronic kidney disease, hypertension, diabetes, and asthma. During a routine follow-up clinic visit, the following labs were noted: Na 142 meq/L, K 5.4 meq/L, Cl 105 meq/L, CO2¬ 25 meq/L, BUN 20 mg/dl, Cr 2.5 mg/dl, Hgb 9.3 Gm/dl, Hct 28%, MCV 82 µm3, Alb 2.4 Gm/dl, Ca 10.5 mg/dl, phosphate 5.5 mg/dl, Fe 90 mcg/dl, TIBC 460 µg/dl, Transferrin saturation 14%, Ferritin 15 ng/ml. The patient has no specific complaints but his vital signs are as follows: BP 170/100 mm Hg, P 82, RR16, T 37°C. Patient is 6’0” and 185 lbs.

Three months after initiation of anti-hypertensive therapy, FJ’s serum creatinine has increased to 3.0 mg/dl. Based only on this new information which of the following is the most appropriate response to this observation?
a. increase dose of anti-hypertensive agent
b. decrease dose of anti-hypertensive agent
c. discontinue present agent; begin therapy with an alternative agent
d. make no change in antihypertensive therapy
d. make no change in antihypertensive therapy

Selection “d” is correct; please see objectives 6 and 7. Such an increase in Cr is well within the 30 to 40% increase that can occur (see slide #37). Since there is no information about the patient’s serum K+ at this time(three months prior it was slightly elevated), there is insufficient information at present to justify any of the alternative strategies listed in selections a thru c. One of these changes might be appropriate if the patient’s blood pressure were known at this time.
2. Which of the following are monitoring parameters for ACE inhibitors?

a. Potassium
b. Creatinine
c. Heart rate
d. A and B
e. A, B, and C
d. A and B

ACE inhibitors do not alter heart rate, but they can increase potassium and creatinine, both of which need to be monitored.
3. Which of the following best describes appropriate use of metoprolol tartrate (immediate release) vs. metoprolol succinate (controlled release) in this patient?

a. Metoprolol tartrate is preferred because it is available generic
b. Metoprolol tartrate should not be used because carvedilol was shown to be superior
c. Either agent may be used in the treatment of heart failure
d. Neither metoprolol tartrate nor metoprolol succinate should be used because carvedilol is preferred
b. Metoprolol tartrate should not be used because carvedilol was shown to be superior

MERIT-HF studied metoprolol succinate and found it to improve mortality in HF. Furthermore, the COMET study compared carvedilol and metoprolol tartrate and found carvedilol to be superior. Therefore, either carvedilol or metoprolol succinate are the preferred beta-blockers in heart failure.
4. QS is a 35 year-old white man with stage 1 primary hypertension. He is overweight, has impaired fasting glucose, and smokes, but is otherwise healthy. He is resistant to dietary and exercise interventions. What is the best drug choice for initial therapy for this patient?
a. triamterene/ hydrochlorothiazide
b. atenolol
c. valsartan
d. trandolapril/ verapamil
c. valsartan

a. Incorrect because the patient is likely to have high-renin hypertension, and the adverse metabolic effects may not be favorable given that he has impaired fasting glucose
b. Incorrect because β-blockers are not the best choice for initial therapy of uncomplicated hypertension in the setting of obesity due to the potential for adverse metabolic effects.
c. Correct because valsartan would be an appropriate initial drug choice for this patient, based on the fact that he is younger and white and may have a renin component, and also because ARBs have beneficial metabolic effects.
d. Incorrect because 2 drug therapy is not necessarily indicated given that he has stage 1 hypertension.
5. Digoxin therapy for congestive heart failure will be started in a 62-year-old, 78-kg man with a stable serum creatinine concentration of 1.8 mg/dl. The patient is not obese (i.e., assume body weight is ideal). Calculate a maintenance dose regimen to achieve a target steady-state concentration at the LOWEST END of the reported therapeutic range for patients with heart failure. Select the regimen that is CLOSEST to the calculated value. Assume a tablet formulation that has a bioavailability of 75% will be used.
CLcr = [(140 – age)  body weight] / (Scr  72)]
Digoxin CL (L/hr) = 3.00 + (0.0546  CLcr ml/min)
MD = [(Target Css  CL ) / F] where  = 24 hours and F = 0.75.

a. 0.089 mg every day
b. 0.107 mg every day
c. 0.125 mg every day
d. 0.142 mg every day
e. 0.178 mg every day
a. 0.089 mg every day

Answer A is correct, calculated as follows:
CLcr = [(140 – 62) x 78] / (1.8 x 72)] = 46.9 ml/min
Digoxin CL = 3.0 + (0.0546 x 46.9) = 5.56 L/hr
Target concentration = 0.50 ng/ml or 0.50 mcg/L
MD/tau = (Css x CL) / F
MD = [(Css x CL x tau)/ F] where F = 0.75 and tau= 24 hrs
MD = [(Target Css x CL x tau) / F] = [(0.50 mcg/L x 5.56 L/hr x 24 hr) / 0.75] = 89 mcg or 0.089 mg/day.
KK is a 45 year old man who presents to his local emergency room with sub-sternal chest pain and is subsequently found to have an STEMI. Therapy with tenecteplase (TNKase) is planned. Which of the following would be an ABSOLUTE contraindication to use of this agent in this patient?
a. blood pressure >180/110 mmHg
b. history of a gastrointestinal bleed 3 weeks ago
c. history of a cerebral hemorrhage 2 years ago
d. history of heparin-induced thrombocytopenia
c. history of a cerebral hemorrhage 2 years ago

Selection “c” is correct; please see objectives 2 and 5. Any cerebral hemorrhage represents an absolute contraindication to any thrombolytic agent regardless of how long ago it occurred. The remainder of the selections are relative contraindications. Please see slide #11 of ACS-II presentation.
Which of the following strategies would be the most INCORRECT in a patient who reports gaining 7 pounds in the last 4 days despite taking their furosemide and other heart failure medications as directed?
a. Add metolazone to furosemide until baseline weight is achieved
b. Increase the patient’s dose of furosemide until baseline weight is achieved
c. Increase the patient’s dose of bisoprolol until baseline weight is achieved
d. Switch the patient from furosemide to torsemide until baseline weight is achieved
c. Increase the patient’s dose of bisoprolol until baseline weight is achieved

The addition of metolazone will enhance natriuresis and diuresis (“sequential nephron blockade”) and would be an acceptable intervention. Increasing the patient’s furosemide dose or switching furosemide to torsemide (a more bioavailable loop diuretic) would be expected to facilitate weight loss. Either of these strategies could be implemented. Increasing the patient’s beta blocker could be detrimental since the patient is currently not stable as evidenced by the recent 7 pound weight gain. See objectives 2, 4
LK is a 46 year old African American male who was recently diagnosed with hypertension. His average BP is 152/102 mmHg. He has no other significant past medical history. Based on the JNC 7 guidelines, and principles of hypertension pathophysiology, which of the following would be the most appropriate initial therapy for LK?

a. Lifestyle modifications for 6 months
b. HCTZ
c. HCTZ + verapamil
d. HCTZ + lisinopril
d. HCTZ + lisinopril

a. INCORRECT - While it is recommended that lifestyle modifications be implemented in all, it is not recommended to do this only, particularly in a patient with Stage 2 HTN, as this patient.
b. INCORRECT – this patient has Stage 2 HTN and it is recommended to initiate therapy with 2 drugs, since 1 drug is unlikely to lead to adequate BP reduction
c. INCORRECT – The best combinations (and the ones marketed commercially) all include drugs that address the “two sides” of the pathophysiology. This combination, a thiazide and CCB would only address one side.
d. CORRECT – 2 drug therapy is recommended for Stage 2 HTN; a combination of a thiazide and ACE inhibitor is highly effective, independent of race. In combination therapy it is best to pair (and the commercially available combination products always pair) a thiazide or CCB with a BB, ACEI or ARB. You don’t see thiazide + CCB combos nor do you see BB + ACEI or ARB combos, because these are not effective combinations. See objective 2.
LL started antihypertensive therapy one month ago, and is in for a follow-up visit. The clinician is going to check LL’s laboratory values. In particular, the clinician wants to monitor for any changes in: glucose, uric acid, potassium and triglycerides. Which antihypertensive drug can cause abnormalities in all of these laboratory parameters, and thus is the drug LL is MOST LIKELY taking?

a. metoprolol
b. chlorthalidone
c. diltiazem
d. candesartan
b. chlorthalidone

a. INCORRECT – metoprolol can cause changes in all the laboratory parameters but is not associated with significant changes in potassium
b. CORRECT – the thiazide diuretics can cause significant changes in all these laboratory parameters
c. INCORRECT – diltiazem is not associated with changes in any of these laboratory parameters
d. INCORRECT - candesartan is not associated with changes in any of these laboratory parameters except potassium See objective 6.
RT is a 74 year old white female with systolic heart failure. The presumed
etiology of her heart failure is hypertension, and her medical history is otherwise
insignificant. The physician judges her to have Class II heart failure
Current medications:
Enalapril (Vasotec) 10 mg bid
Digoxin (Digitek, Lanoxin) 0.125 mg qd
furosemide (Lasix) 20 mg qd
KCl (potassium chloride) – 10 mEq qd.
Vital Signs: BP – 94/52 mmHg P – 56/min R – 14/min
Regarding the initiation of β-blocker therapy, which of the following would be
most appropriate for this patient?
a. Initiation of a β-blocker is inappropriate in this patient at this time
b. Recommend using metoprolol (Lopressor, Toprol XL) over carvedilol
(Coreg) because metoprolol doesn’t have alpha-blocking effects, which are
a concern in this elderly patient.
c. Recommend initiation of metoprolol because it has less blood pressure
lowering effects than carvedilol
d. Recommend decreasing the dose of enalapril, then initiating metoprolol.
a. Initiation of a β-blocker is inappropriate in this patient at this time

While beta-blocker is strongly indicated in all heart failure patients, it is
contraindicated in bradycardia and hypotension. The main issue of concern here is
the bradycardia, secondarily the relatively low BP. It might be possible to get the BP
up by reducing the enalapril dose and might be possible to get the HR up by stopping
the digoxin. Since betab-blockers have mortality benefit and digoxin does not, this
would be acceptable. Alternatively some would insert a pacemaker in bradycardic patients to be able to use a beta-blocker.
RH is a 54 year old white male with a 10 year history of hypertension. His blood
pressure is currently controlled with lisinopril 20 mg qd. He recently has
developed symptoms of chest pain with exertion, and has just been diagnosed with
chronic stable angina. He presently has anginal symptoms about 4 to 5 times per
week. Among the following, which is the most rational approach to management
of RH?
a. add SL nitroglycerin
b. add diltiazem (Cardizem, Diltia, others) and SL nitroglycerin
c. add nifedipine SR (Adalta, Procardia, others) and SL nitroglyercin and
d/c lisinopril (Prinvil, Zestril)
d. add metoprolol and SL nitroglycerin and d/c lisinopril
b. add diltiazem (Cardizem, Diltia, others) and SL nitroglycerin

a is incorrect because the patient has frequent symptoms that warrant preventive
therapy versus just abortive therapy. b is correct because this adds both a preventive
and abortive agent. c is incorrect because dihydropyridines should not be used as
monotherapy for prevention and also because the lisinopril shouldn’t be d/c’d since
ACEI’s have substantial benefits in risk reduction for individuals with coronary
3
artery disease. d is incorrect because of the recommendation to d/c lisinopril. Otherwise the recommendations in d are fine.
CC is a 62 year old white male with moderate asthma, hypertension and new onset
atrial fibrillation. Which of the following is the most appropriate antihypertensive
therapy for CC?
a. atenolol (Tenormin)
b. diltiazem (Cardizem)
c. captopril (Capoten)
d. chlorthalidone (thalitone)
b. diltiazem (Cardizem)

In this case you want drugs that will help control HTN and AF, and not be adverse in
asthma. a is incorrect because it has the potential to worsen asthma, even though it
would be beneficial in the AF. b is correct because it is not adverse in asthma and
has rate control properties for AF. c is incorrect because it doesn’t have any benefits
for rate control in AF and an ACEI cough may exacerbate asthma. d is incorrect
because it doesn’t have any rate slowing properties in AF.
BJ is a 42 year old African American female recently diagnosed with hypertension.
Her past medical history is otherwise unremarkable. She has been following a
sodium restricted diet for the past 3 months. Her blood pressure is now averaging
152/98 mmHg. Which of the following would be the most preferred agent for
management of her hypertension?
a. HCTZ (Esidrix, others)
b. atenolol (Tenormin)
c. enalapril (Vasotec)
d. amlodipine (Norvasc)
a. HCTZ (Esidrix, others)

Any of the choices would be considered acceptable first line therapy. a is considered
prefered in this patient at several levels. First, thiazides are considered preferred in
JNC 7 for uncomplicated HTN. Also it is likely to be effective in this black
hypertensive. b and c are not considered preferred in JNC 7 and also would be less
likely to work in this patient. d is likely to work, but not considered preferred under
JNC 7 guidelines.
TT is a 54 year old male who was released from the hospital 6 weeks ago after
having an acute myocardial infarction. His lipid panel is TC=250 mg/dl;
LDL=120 mg/dl; HDL=45 mg/dl; TG=150 mg/dl. Which of the following is the
best option for TT?
a. Pravastatin 40 mg qd
b. Atorvastatin (Lipitor) 10 mg qd
c. Atorvastatin 80 mg qd
d. Simvastatin (Zocor) 20 mg qd
c. Atorvastatin 80 mg qd

Recent ACS puts this patient in the highest risk category with and LDL goal of <70
mg/dl. Furthermore, in clinical studies atorvastatin 80 mg has been shown to be
superior in event reduction compared with low to moderate statin doses in stable CVD and ACS. Although a, b, and d are reasonable, (c) is the best option.
Refer to Case G
Consideration is being given to starting this patient on spironolactone. Which of the following is the most appropriate response to this suggestion?
a. Spironolactone (and eplerenone) should be avoided in this patient
b. Given patient’s class III heart failure status and recent exacerbation, spironolactone is indicated and should be started
c. Eplerenone should be recommended since spironolactone could be problematic in this patient
d. This patient has no indication for spironolactone
e. Spironolactone has only been studied in African Americans so its benefits in this patient are unclear, thus should probably be avoided.
a. Spironolactone (and eplerenone) should be avoided in this patient

While this patient is a good candidate in general for an aldosterone antagonist, they should be avoided in him, due to his poor renal function and high K. It is not true that spironolactone has only been studied in African Americans, and if this patient had low K and good renal function, he might be a reasonable candidate for such therapy.